K
Khách

Hãy nhập câu hỏi của bạn vào đây, nếu là tài khoản VIP, bạn sẽ được ưu tiên trả lời.

AH
Akai Haruma
Giáo viên
6 tháng 7 2019

Bạn tham khảo lời giải bài 4 link sau:

Câu hỏi của Bonking - Toán lớp 9 | Học trực tuyến

7 tháng 7 2019

Bài giải rất hay, mình cảm ơn bạn nhiều

11 tháng 11 2019

2/ Không mất tính tổng quát, giả sử \(c=min\left\{a,b,c\right\}\).

Nếu abc = 0 thì có ít nhất một số bằng 0. Giả sử c = 0. BĐT quy về: \(a^2+b^2\ge2ab\Leftrightarrow\left(a-b\right)^2\ge0\) (luôn đúng)

Đẳng thức xảy ra khi a = b; c = 0.

Nếu \(abc\ne0\). Chia hai vế của BĐT cho \(\sqrt[3]{\left(abc\right)^2}\)

BĐT quy về: \(\Sigma_{cyc}\sqrt[3]{\frac{a^4}{b^2c^2}}+3\ge2\Sigma_{cyc}\sqrt[3]{\frac{ab}{c^2}}\)

Đặt \(\sqrt[3]{\frac{a^2}{bc}}=x;\sqrt[3]{\frac{b^2}{ca}}=y;\sqrt[3]{\frac{c^2}{ab}}=z\Rightarrow xyz=1\)

Cần chúng minh: \(x^2+y^2+z^2+3\ge2\left(\frac{1}{x}+\frac{1}{y}+\frac{1}{z}\right)\)

\(\Leftrightarrow x^2+y^2+z^2+2xyz+1\ge2\left(xy+yz+zx\right)\) (1)

Theo nguyên lí Dirichlet thì trong 3 số x - 1, y - 1, z - 1 tồn tại ít nhất 2 số có tích không âm. Không mất tính tổng quát, giả sử \(\left(x-1\right)\left(y-1\right)\ge0\)

\(\Rightarrow2xyz\ge2xz+2yz-2z\). Thay vào (1):

\(VT\ge x^2+y^2+z^2+2xz+2yz-2z+1\)

\(=\left(x-y\right)^2+\left(z-1\right)^2+2xy+2xz+2yz\)

\(\ge2\left(xy+yz+zx\right)\)

Vậy (1) đúng. BĐT đã được chứng minh.

Đẳng thức xảy ra khi a = b = c hoặc a = b, c = 0 và các hoán vị.

Check giúp em vs @Nguyễn Việt Lâm, bài dài quá:(

6 tháng 7 2020

Để đưa về chứng minh $(1)$ và $(2)$ ta dùng:

Định lí SOS: Nếu \(X+Y+Z=0\) thì \(AX^2+BY^2+CZ^2\ge0\)

khi \(\left\{{}\begin{matrix}A+B+C\ge0\\AB+BC+CA\ge0\end{matrix}\right.\)

Chứng minh: Vì \(\sum\left(A+C\right)=2\left(A+B+C\right)\ge0\)

Nên ta có thể giả sử \(A+C\ge0\). Mà $X+Y+Z=0$ nên$:$

\(AX^2+BY^2+CZ^2=AX^2+BY^2+C\left[-\left(X+Y\right)\right]^2\)

\(={\frac { \left( AX+CX+CY \right) ^{2}}{A+C}}+{\frac {{Y}^{2} \left( AB+AC+BC \right) }{A+C}} \geq 0\)

22 tháng 8 2020

Cái phần CMR:   \(\left(a-1\right)^2+\left(b-2\right)^2+\left(c-3\right)^2\le3\left(b-2\right)\)     phải là giả thiết chứ nhỉ ??

ĐỀ GỐC BÀI NÀY LÀ ĐỀ CỦA CHUYÊN HƯNG YÊN NHÉ, THẦY CẬU RA LẠI THÔI !!!!!

DO:   \(a\ge1;b\ge2;c\ge3\Rightarrow a-1;b-2;c-3\ge0\)

ĐẶT:   \(a-1=x;b-2=y;c-3=z\)

=>     \(gt\Leftrightarrow\hept{\begin{cases}x;y;z\ge0\\x^2+y^2+z^2\le3y\end{cases}}\)

=>   \(a=x+1;b=y+2;c=z+3\)

=>   \(P=\frac{1}{\left(x+1\right)^2}+\frac{4}{\left(y+2\right)^2}+\frac{8}{\left(z+3\right)^2}\)

TA ÁP DỤNG 2 BĐT SAU:   \(\hept{\begin{cases}\left(x+1\right)^2\le2\left(x^2+1\right)\\\left(z+3\right)^2\le4\left(z^2+3\right)\end{cases}}\)

=>   \(P\ge\frac{1}{2\left(x^2+1\right)}+\frac{8}{4\left(z^2+3\right)}+\frac{4}{\left(y+2\right)^2}\)

=>   \(P\ge\frac{1}{2\left(x^2+1\right)}+\frac{4}{2\left(z^2+3\right)}+\frac{4}{\left(y+2\right)^2}\)

=>   \(P\ge\frac{\left(1+2\right)^2}{2\left(x^2+z^2\right)+8}+\frac{4}{\left(y+2\right)^2}\)      (BĐT CAUCHY - SCHWARZ)

=>   \(P\ge\frac{9}{2\left(x^2+z^2\right)+8}+\frac{4}{\left(y+2\right)^2}\)

MÀ:   \(x^2+z^2\le3y-y^2\)     (gt)

=>   \(P\ge\frac{9}{2\left(3y-y^2\right)}+\frac{4}{\left(y+2\right)^2}=\frac{9}{6y-2y^2}+\frac{4}{\left(y+2\right)^2}\)

TA SẼ CHỨNG MINH    \(\frac{9}{6y-2y^2+8}+\frac{4}{\left(y+2\right)^2}\ge1\)

<=>   \(\left(y-2\right)^2\left(2y^2+10y+9\right)\ge0\)         (*)

 (CHỖ NÀY CẬU QUY ĐỒNG MẪU SỐ, RÚT GỌN RỒI PHÂN TÍCH NHÂN TỬ SẼ RA ĐƯỢC NHƯ THẾ NÀY, MÌNH LÀM TẮT NHA)

DO:   \(\hept{\begin{cases}\left(y-2\right)^2\ge0\forall y\\2y^2+10y+9\ge9>0\left(y\ge0\right)\end{cases}}\)

VẬY BĐT (*) LUÔN ĐÚNG !!!!!!

=>   \(P\ge1\)

DẤU "=" XẢY RA <=>   \(x=z=1;y=2\)

<=>   \(a=2;b=4;c=4\)

22 tháng 8 2020

Đề cho vậy đó, bn CM cái "giả thiết" giúp mk với:)

30 tháng 4 2020

\(P=\frac{bc}{2ab+ac}+\frac{ca}{2ab+bc}+\frac{4ab}{bc+ca}\)

Xét \(Q=P+3=\frac{bc}{2ab+ac}+1+\frac{ca}{2ab+bc}+1+\frac{4ab}{bc+ca}+1\)

\(Q=\frac{2ab+ac+bc}{2ab+ac}+\frac{2ab+ac+bc}{2ab+bc}+\frac{4ab+bc+ca}{bc+ca}\)

\(=\left(2ab+ac+bc\right)\left(\frac{1}{2ab+ac}+\frac{1}{2ab+bc}\right)+\frac{4ab+bc+ca}{bc+ca}\)

\(\ge\left(2ab+ac+bc\right)\frac{4}{4ab+ac+bc}+\frac{4ab+bc+ca}{bc+ca}=K\)(Áp dụng BĐT \(\frac{1}{a}+\frac{1}{b}\ge\frac{4}{a+b}\)với a, b không âm)

\(K=\frac{2\left(4ab+ac+bc\right)+2\left(ac+bc\right)}{4ab+ac+bc}+\frac{2\left(4ab+bc+ca\right)}{9\left(ac+bc\right)}\)\(+\frac{7\left(4ab+bc+ca\right)}{9\left(ac+bc\right)}\)

\(=2+\left[\frac{2\left(ac+bc\right)}{4ab+ac+bc}+\frac{2\left(4ab+bc+ca\right)}{9\left(ac+bc\right)}\right]+\frac{7}{9}+\frac{7}{9}.\frac{4ab}{ac+bc}\)

\(\ge2+2\sqrt{\frac{2\left(ac+bc\right)}{4ab+ac+bc}.\frac{2\left(4ab+bc+ca\right)}{9\left(ac+bc\right)}}+\frac{7}{9}+\frac{7}{9}.\frac{4ab}{ac+bc}\)(Áp dụng BĐT Cô - si cho 2 số không âm)

\(=\frac{37}{9}+\frac{7}{9}.\frac{4ab}{ac+bc}\)

Mặt khác: \(6=2\left(\frac{a}{b}+\frac{b}{a}\right)+c\left(\frac{a}{b^2}+\frac{b}{a^2}\right)=\frac{2\left(a^2+b^2\right)}{ab}+\frac{c\left(a^3+b^3\right)}{a^2b^2}\)

\(=\frac{2\left(a^2+b^2\right)}{ab}+\frac{c\left(a+b\right)\left(a^2-ab+b^2\right)}{a^2b^2}\)\(\ge\frac{2.2ab}{ab}+\frac{c\left(a+b\right)\left(2ab-ab\right)}{a^2b^2}=4+\frac{ac+bc}{ab}\)(theo BĐT \(a^2+b^2\ge2ab\))

\(\Rightarrow\frac{ac+bc}{ab}\le2\Leftrightarrow\frac{ab}{ac+bc}\ge\frac{1}{2}\)

\(\Rightarrow K\ge\frac{37}{9}+\frac{7}{9}.\frac{4ab}{ac+bc}\ge\frac{37}{9}+\frac{7}{9}.\frac{4}{2}=\frac{17}{3}\)

Ta có \(Q=P+3\ge K\ge\frac{17}{3}\Rightarrow P\ge\frac{17}{3}-3=\frac{8}{3}\)

Đẳng thức xảy ra khi \(\hept{\begin{cases}2ab+ac=2ab+bc\\\frac{2\left(ac+bc\right)}{4ab+ac+bc}=\frac{2\left(4ab+bc+ca\right)}{9\left(ac+bc\right)}\\a=b\end{cases}}\)\(\Leftrightarrow a=b=c\)

1 tháng 5 2020

Từ \(2\left(\frac{a}{b}+\frac{b}{a}\right)+c\left(\frac{a}{b^2}+\frac{b}{a^2}\right)=6\Rightarrow6=\frac{c\left(a+b\right)\left(a^2-ab+b^2\right)}{a^2b^2}+\frac{2\left(a^2+b^2\right)}{ab}\)

ta có \(a^2+b^2\ge2ab\Rightarrow6=\frac{c\left(a+b\right)\left(a^2-ab+b^2\right)}{a^2b^2}+\frac{2\left(a^2+b^2\right)}{ab}\ge\frac{c\left(a+b\right)}{ab}+4\)

\(\Rightarrow0< \frac{c\left(a+b\right)}{ab}\le2\)

Lại có 

\(\frac{bc}{a\left(2b+c\right)}+\frac{ac}{b\left(2a+c\right)}=\frac{\left(bc\right)^2}{abc\left(2b+c\right)}+\frac{\left(ac\right)^2}{abc\left(2a+c\right)}\ge\frac{\left(bc+ac\right)^2}{2abc\left(a+b+c\right)}\)\(=\frac{\left[c\left(a+b\right)\right]^2}{2abc\left(a+b+c\right)}\)

và \(abc\left(a+b+c\right)=ab\cdot bc+bc\cdot ba+ab\cdot ca\le\frac{\left(ab+bc+ca\right)^2}{3}\)

\(\Rightarrow\frac{bc}{a\left(2b+c\right)}+\frac{ac}{b\left(2a+c\right)}\ge\frac{3}{2}\left(\frac{c\left(a+b\right)}{ab+bc+ca}\right)^2=\frac{3}{2}\left(\frac{\frac{c\left(a+b\right)}{ab}}{1+\frac{c\left(a+b\right)}{ab}}\right)^2\)

Đặt \(t=\frac{c\left(a+b\right)}{ab}\Rightarrow P\ge\frac{3t^2}{2\left(1+t\right)^2}+\frac{4}{t}\left(0< t\le2\right)\)

Có \(\frac{3t^2}{2\left(1+t\right)^2}+\frac{4}{t}=\left(\frac{3t^2}{\left(1+t\right)^2}+\frac{4}{t}-\frac{8}{3}\right)+\frac{8}{3}=\frac{-7t^2-8t^2+32t+24}{6t\left(1+t\right)^2}+\frac{8}{3}\)

\(=\frac{\left(t-2\right)\left(-7t^2-22t-12\right)}{6t\left(1+t\right)^2}\ge0\forall t\in(0;2]\)

=> \(\frac{\left(t-2\right)\left(-7t^2-22t-12\right)}{6t\left(1+t\right)^2}+\frac{8}{3}\ge\frac{8}{3}\forall t\in(0;2]\frac{1}{2}\)

Dấu "=" xảy ra <=> t=2 hay a=b=c

27 tháng 10 2020

Bài 2: Ta có: x, y, z không âm và \(x+y+z=\frac{3}{2}\)nên \(0\le x\le\frac{3}{2}\Rightarrow2-x>0\)

Áp dụng bất đẳng thức AM - GM dạng \(ab\le\frac{\left(a+b\right)^2}{4}\), ta được: \(x+2xy+4xyz=x+4xy\left(z+\frac{1}{2}\right)\le x+4x.\frac{\left(y+z+\frac{1}{2}\right)^2}{4}=x+x\left(2-x\right)^2\)

Ta cần chứng minh \(x+x\left(2-x\right)^2\le2\Leftrightarrow\left(2-x\right)\left(x-1\right)^2\ge0\)*đúng*

Đẳng thức xảy ra khi \(\left(x,y,z\right)=\left(1,\frac{1}{2},0\right)\)

29 tháng 10 2020

Bài 3: Áp dụng đánh giá quen thuộc \(4ab\le\left(a+b\right)^2\), ta có: \(2\le\left(x+y\right)^3+4xy\le\left(x+y\right)^3+\left(x+y\right)^2\)

Đặt x + y = t thì ta được: \(t^3+t^2-2\ge0\Leftrightarrow\left(t-1\right)\left(t^2+2t+2\right)\ge0\Rightarrow t\ge1\)(dễ thấy \(t^2+2t+2>0\forall t\))

\(\Rightarrow x^2+y^2\ge\frac{\left(x+y\right)^2}{2}\ge\frac{1}{2}\)

\(P=3\left(x^4+y^4+x^2y^2\right)-2\left(x^2+y^2\right)+1=3\left[\frac{3}{4}\left(x^2+y^2\right)^2+\frac{1}{4}\left(x^2-y^2\right)^2\right]-2\left(x^2+y^2\right)+1\ge\frac{9}{4}\left(x^2+y^2\right)^2-2\left(x^2+y^2\right)+1\)\(=\frac{9}{4}\left[\left(x^2+y^2\right)^2+\frac{1}{4}\right]-2\left(x^2+y^2\right)+\frac{7}{16}\ge\frac{9}{4}.2\sqrt{\left(x^2+y^2\right)^2.\frac{1}{4}}-2\left(x^2+y^2\right)+\frac{7}{16}=\frac{9}{4}\left(x^2+y^2\right)-2\left(x^2+y^2\right)+\frac{7}{16}=\frac{1}{4}\left(x^2+y^2\right)+\frac{7}{16}\ge\frac{1}{8}+\frac{7}{16}=\frac{9}{16}\)Đẳng thức xảy ra khi x = y = 1/2

6 tháng 5 2017

BĐT cần chứng minh tương đương \(a^4+b^4+c^4\ge2\left(a^2b^2+b^2c^2+c^2a^2\right)-abc\left(a+b+c\right)\)

mà \(a^2b^2+b^2c^2+c^2a^2\ge abc\left(a+b+c\right)\)(BĐT cauchy)

\(\Leftrightarrow a^4+b^4+c^4\ge a^2b^2+b^2c^2+c^2a^2\)(cần chứng minh)

ÁP dụng bất đẳng thức bunyakovsky:

\(3\left(a^4+b^4+c^4\right)\ge\left(a^2+b^2+c^2\right)^2\)

mà \(\left(a^2+b^2+c^2\right)^2\ge3\left(a^2b^2+b^2c^2+c^2a^2\right)\)(hệ quả BĐT cauchy)

\(\Rightarrow3\left(a^4+b^4+c^4\right)\ge3\left(a^2b^2+b^2c^2+c^2a^2\right)\)

\(\Leftrightarrow a^4+b^4+c^4\ge a^2b^2+b^2c^2+c^2a^2\)(đpcm)

dấu = xảy ra khi a=b=c

7 tháng 5 2017

Trái dấu bất đẳng thức rồi kìa